Bạn chưa đăng nhập. Vui lòng đăng nhập để hỏi bài
Nguyễn Trung Hiếu


Cho a,b,c là các số dương . Chứng minh rằng:
a) \(A=\frac{a}{2a+b+c}+\frac{b}{2b+c+a}+\frac{c}{2c+a+b}\le\frac{3}{4}\)
b) \(B=\left(a^5-a^2+3\right)\left(b^5-b^2+3\right)\left(c^5-c^2+3\right)\ge\left(a+b+c\right)^3\)
 

Thắng Nguyễn
8 tháng 2 2017 lúc 18:37

Áp dụng C-S

\(\frac{a}{2a+b+c}=\frac{a}{\left(a+b\right)+\left(a+c\right)}\le\frac{a}{4}\left(\frac{1}{a+b}+\frac{1}{a+c}\right)\)

Trần Quốc Đạt
9 tháng 2 2017 lúc 6:03

b) chính là USAMO 2004. Đây là lời giải cung cấp bởi "http://www.artofproblemsolving.com/wiki/index.php/2004_USAMO_Problems/Problem_5"

Ta chứng minh được \(x^5+1\ge x^3+x^2\) suy ra \(x^5-x^2+3\ge x^3+2\).

Ta chỉ cần CM được \(\left(a^3+1+1\right)\left(1+b^3+1\right)\left(1+1+c^3\right)\ge\left(a+b+c\right)^3\)

Nhưng đây chính là BĐT Holder cho 3 bộ số mỗi bộ 3 số.


Các câu hỏi tương tự
Lê Minh Đức
Xem chi tiết
Trần Lâm Thiên Hương
Xem chi tiết
hhhhh
Xem chi tiết
Mai Tuấn Hưng
Xem chi tiết
Nguyễn Thị Minh Nguyệt
Xem chi tiết
Phạm Văn Việt
Xem chi tiết
Phùng Gia Bảo
Xem chi tiết
Nguyễn Thiều Công Thành
Xem chi tiết
Nguyễn Lâm Ngọc
Xem chi tiết